Difference between revisions of "1980 AHSME Problems/Problem 9"

m (Problem)
(Solution)
Line 6: Line 6:
  
 
== Solution ==
 
== Solution ==
 +
Let us think about this. We only know
 
<math>\fbox{}</math>
 
<math>\fbox{}</math>
  

Revision as of 15:19, 22 April 2016

Problem

A man walks $x$ miles due west, turns $150^\circ$ to his left and walks 3 miles in the new direction. If he finishes a a point $\sqrt{3}$ from his starting point, then $x$ is

$\text{(A)} \ \sqrt 3 \qquad \text{(B)} \ 2\sqrt{5} \qquad \text{(C)} \ \frac 32 \qquad \text{(D)} \ 3 \qquad \text{(E)} \ \text{not uniquely determined}$

Solution

Let us think about this. We only know $\fbox{}$

See also

1980 AHSME (ProblemsAnswer KeyResources)
Preceded by
Problem 8
Followed by
Problem 10
1 2 3 4 5 6 7 8 9 10 11 12 13 14 15 16 17 18 19 20 21 22 23 24 25 26 27 28 29 30
All AHSME Problems and Solutions

The problems on this page are copyrighted by the Mathematical Association of America's American Mathematics Competitions. AMC logo.png